Found problems: 85335
1960 Putnam, A6
A player repeatedly throwing a die is to play until their score reaches or passes a total $n$. Denote by $p(n)$ the probability of making exactly the total $n,$ and find the value of $\lim_{n \to \infty} p(n).$
1986 AMC 12/AHSME, 5
Simplify $\left(\sqrt[6]{27} - \sqrt{6 \frac{3}{4} }\right)^2$
$ \textbf{(A)}\ \frac{3}{4} \qquad
\textbf{(B)}\ \frac{\sqrt 3}{2} \qquad
\textbf{(C)}\ \frac{3 \sqrt 3}{4} \qquad
\textbf{(D)}\ \frac{3}{2} \qquad
\textbf{(E)}\ \frac{3 \sqrt 3}{2} $
2012 QEDMO 11th, 7
In the following, a rhombus is one with edge length $1$ and interior angles $60^o$ and $120^o$ . Now let $n$ be a natural number and $H$ a regular hexagon with edge length $n$, which is covered with rhombuses without overlapping has been. The rhombuses then appear in three different orientations. Prove that whatever the overlap looks exactly, each of these three orientations can be viewed at the same time.
2025 Austrian MO Regional Competition, 4
Let $z$ be a positive integer that is not divisible by $8$. Furthermore, let $n \geqslant 2$ be a positive integer. Prove that none of the numbers of the form $z^n + z + 1$ is a square number.
[i](Walther Janous)[/i]
2024 Serbia JBMO TST, 4
Let $I$ be the incenter of a triangle $ABC$ with $AB \neq AC$. Let $M$ be the midpoint of $BC$, $M' \in BC$ be such that $IM'=IM$ and $K$ be the midpoint of the arc $BAC$. If $AK \cap BC=L$, show that $KLIM'$ is cyclic.
2008 Balkan MO Shortlist, G1
In acute angled triangle $ABC$ we denote by $a,b,c$ the side lengths, by $m_a,m_b,m_c$ the median lengths and by $r_{b}c,r_{ca},r_{ab}$ the radii of the circles tangents to two sides and to circumscribed circle of the triangle, respectively. Prove that
$$\frac{m_a^2}{r_{bc}}+\frac{m_b^2}{r_{ab}}+\frac{m_c^2}{r_{ab}} \ge \frac{27\sqrt3}{8}\sqrt[3]{abc}$$
2014 ASDAN Math Tournament, 4
A frog is hopping from $(0,0)$ to $(8,8)$. The frog can hop from $(x,y)$ to either $(x+1,y)$ or $(x,y+1)$. The frog is only allowed to hop to point $(x,y)$ if $|y-x|\leq1$. Compute the number of distinct valid paths the frog can take.
VMEO III 2006 Shortlist, N3
Given odd prime $p$. Sequence ${x_n}$ is defined by $x_{n+2}= 4x_{n+1}-x_n$.
Choose $x_0,x_1$ such that for every random positive integer $k$, there exists $i\in \mathbb N$ such that $4p^2-8p+1|x_i - (2p)^k$.
2022 South East Mathematical Olympiad, 5
Positive sequences $\{a_n\},\{b_n\}$ satisfy:$a_1=b_1=1,b_n=a_nb_{n-1}-\frac{1}{4}(n\geq 2)$.
Find the minimum value of $4\sqrt{b_1b_2\cdots b_m}+\sum_{k=1}^m\frac{1}{a_1a_2\cdots a_k}$,where $m$ is a given positive integer.
2010 IMAR Test, 1
Show that a sequence $(a_n)$ of $+1$ and $-1$ is periodic with period a power of $2$ if and only if $a_n=(-1)^{P(n)}$, where $P$ is an integer-valued polynomial with rational coefficients.
2010 Tournament Of Towns, 5
$N$ horsemen are riding in the same direction along a circular road. Their speeds are constant and pairwise distinct. There is a single point on the road where the horsemen can surpass one another. Can they ride in this fashion for arbitrarily long time? Consider the cases:
$(a) N = 3;$
$(b) N = 10.$
2013 Stanford Mathematics Tournament, 4
Given the digits $1$ through $7$, one can form $7!=5040$ numbers by forming different permutations of the $7$ digits (for example, $1234567$ and $6321475$ are two such permutations). If the $5040$ numbers are then placed in ascending order, what is the $2013^{\text{th}}$ number?
2024 Olympic Revenge, 5
Régis, Ed and Rafael are at the IMO. They are going to play a game in Bath, and there are $2^n$ houses in the city. Régis and Ed will team up against Rafael. The game operates as follows: First, Régis and Ed think on a strategy and then let Rafael know it. After this, Régis and Ed no longer communicate, and the game begins. Rafael decides on an order to visit the houses and then starts taking Régis to them in that order. At each house, except for the last one, Régis choose a number between $1$ and $n$ and places it in the house. In the last house, Rafael chooses a number from $1$ to $n$ and places it there.
Afterwards, Ed sees all the houses and the numbers in them, and he must guess in which house Rafael placed the number. Ed is allowed $k$ guesses. What is the smallest $k$ for which there exists a strategy for Ed and Régis to ensure that Ed correctly guess the house where Rafael placed the number?
2019 AMC 10, 11
Two jars each contain the same number of marbles, and every marble is either blue or green. In Jar 1 the ratio of blue to green marbles is 9:1, and the ratio of blue to green marbles in Jar 2 is 8:1. There are 95 green marbles in all. How many more blue marbles are in Jar 1 than in Jar 2?
$\textbf{(A) } 5 \qquad\textbf{(B) } 10 \qquad\textbf{(C) } 25 \qquad\textbf{(D) } 45 \qquad\textbf{(E) } 50$
2021 239 Open Mathematical Olympiad, 5
Let $a,b,c$ be some complex numbers. Prove that
$$|\dfrac{a^2}{ab+ac-bc}| + |\dfrac{b^2}{ba+bc-ac}| + |\dfrac{c^2}{ca+cb-ab}| \ge \dfrac{3}{2}$$
if the denominators are not 0
MathLinks Contest 4th, 4.2
We say that two triangles $T_1$ and $T_2$ are contained one in each other, and we write $T_1 \subset T_2$, if and only if all the points of the triangle $T_1$ lie on the sides or in the interior of the triangle $T_2$.
Let $\Delta$ be a triangle of area $S$, and let $\Delta_1 \subset \Delta$ be the largest equilateral triangle with this property, and let $\Delta \subset \Delta_2$ be the smallest equilateral triangle with this property (in terms of areas). Let $S_1, S_2$ be the areas of $\Delta_1, \Delta_2$ respectively.
Prove that $S_1S_2 = S^2$.
Bonus question: : Does this statement hold for quadrilaterals (and squares)?
1996 Putnam, 4
$S$ be a set of ordered triples $(a,b,c)$ of distinct elements of a finite set $A$. Suppose that
[list=1]
[*] $(a,b,c)\in S\iff (b,c,a)\in S$
[*] $(a,b,c)\in S\iff (c,b,a)\not\in S$
[*] $(a,b,c),(c,d,a)\text{ both }\in S\iff (b,c,d),(d,a,b)\text{ both }\in S$[/list]
Prove there exists $g: A\to \mathbb{R}$, such that $g$ is one-one and $g(a)<g(b)<g(c)\implies (a,b,c)\in S$
2009 Cuba MO, 2
In Hidro planet were living $2008^2$ hydras some time ago. One of them had 1 tentacle, other 2 and so on to the last with $2008^2$ tentacles. Let $t(H)$ be the number of tentacles of hydra $H$. The pairing of $H_1$ and $H_2$ (where $t(H_1) < t(H_2)$) is a new hydra with $t(H_2)-t(H_1)+8$ tentacles, in case of $t(H_1)=t(H_2)$ they die. An expedition found in Hidro the last hydra with 23 tentacles. That could be true ?
2000 Switzerland Team Selection Test, 4
Let $q(n)$ denote the sum of the digits of a natural number $n$. Determine $q(q(q(2000^{2000})))$.
LMT Speed Rounds, 2011.3
Segment $\overline{PQ}$ is drawn and squares $ABPQ$ and $CDQP$ are constructed in the plane such that they lie on opposite sides of segment $\overline{PQ}.$ If $PQ=1,$ find $BD.$
2009 District Olympiad, 4
Let $ABC$ be an equilateral $ABC$. Points $M, N, P$ are located on the sides $AC, AB, BC$, respectively, such that $\angle CBM= \frac{1}{2} \angle AMN = \frac{1}{3} \angle BNP$ and $\angle CMP = 90 ^o$.
a) Show that $\vartriangle NMB$ is isosceles.
b) Determine $\angle CBM$.
1986 French Mathematical Olympiad, Problem 5
The functions $f,g:[0,1]\to\mathbb R$ are given with the formulas
$$f(x)=\sqrt[4]{1-x},\enspace g(x)=f(f(x)),$$
and $c$ denotes any solution of $x=f(x)$.
(a) i. Analyze the function $f(x)$ and draw its graph. Prove that the equation $f(x)=x$ has the unique root $c$ satisfying $c\in[0.72,0.73]$.
ii. Analyze the function $f'(x)$. Let $M_1$ and $M_2$ be the points of the graph of $f(x)$ with different $x$ coordinates. What is the position of the arc $M_1M_2$ of the graph with respect to the segment $M_1M_2$?
iii. Analyze the function $g(x)$ and draw its graph. What is the position of that graph with respect to the line $y=x$? Find the tangents to the graph at points with $x$ coordinates $0$ and $1$.
iv. Prove that every sequence $\{a_n\}$ with the conditions $a_1\in(0,1)$ and
$a_{n+1}=f(a_n)$ for $n\in\mathbb N$ converges.
[hide=Official Hint]Consider the sequences $\{a_{2n-1}\},\{a_{2n}\}~(n\in\mathbb N)$ and the function $g(x)$ associated with the graph.[/hide]
(b) On the graph of the function $f(x)$ consider the points $M$ and $M'$ with $x$ coordinates $x$ and $f(x)$, where $x\ne c$.
i. Prove that the line $MM'$ intersects with the line $y=x$ at the point with $x$ coordinate
$$h(x)=x-\frac{(f(x)-x)^2}{g(x)+x-2f(x)}.$$
ii. Prove that if $x\in(0,c)$ then $h(x)\in(x,c)$.
iii. Analyze whether the sequence $\{a_n\}$ satisfying $a_1\in(0,c),a_{n+1}=h(a_n)$ for $n\in\mathbb N$ converges. Prove that the sequence $\{\tfrac{a_{n+1}-c}{a_n-c}\}$ converges and find its limit.
(c) Assume that the calculator approximates every number $b\in[-2,2]$ by number $\overline b$ having $p$ decimal digits after the decimal point. We are performing the following sequence of operations on that calculator:
1) Set $a=0.72$;
2) Calculate $\delta(a)=\overline{f(a)}-a$;
3) If $|\delta(a)|>0.5\cdot10^{-p}$, then calculate $\overline{h(a)}$ and go to the operation $2)$ using $\overline{h(a)}$ instead of $a$;
4) If $|\delta(a)|\le0.5\cdot10^{-p}$, finish the calculation.
Let $\overleftrightarrow c$ be the last of calculated values for $\overline{h(a)}$. Assuming that for each $x\in[0.72,0.73]$ we have $\left|\overline{f(x)}-f(x)\right|<\epsilon$, determine $\delta(\overleftrightarrow c)$, the accuracy (depending on $\epsilon$) of the approximation of $c$ with $\overleftrightarrow c$.
(d) Assume that the sequence $\{a_n\}$ satisfies $a_1=0.72$ and $a_{n+1}=f(a_n)$ for $n\in\mathbb N$. Find the smallest $n_0\in\mathbb N$, such that for every $n\ge n_0$ we have $|a_n-c|<10^{-6}$.
1998 Belarus Team Selection Test, 3
Let $s,t$ be given nonzero integers, $(x,y)$ be any (ordered) pair of integers. A sequence of moves is performed as follows: per move $(x,y)$ changes to $(x+t, y-s)$. The pair (x,y) is said to be [i]good [/i] if after some (may be, zero) number of moves described a pair of integers arises that are not relatively prime.
a) Determine whether $(s,t)$ is itself a good pair;
bj Prove that for any nonzero $s$ and $t$ there is a pair $(x,y)$ which is not good.
1989 Romania Team Selection Test, 3
Let $F$ be the boundary and $M,N$ be any interior points of a triangle $ABC$. Consider the function $f_{M,N}: F \to R$ defined by $f_{M,N}(P) = MP^2 +NP^2$ and let $\eta_{M,N}$ be the number of points $P$ for which $f{M,N}$ attains its minimum.
(a) Prove that $1 \le \eta_{M,N} \le 3$.
(b) If $M$ is fixed, find the locus of $N$ for which $\eta_{M,N} > 1$.
(c) Prove that the locus of $M$ for which there are points $N$ such that $\eta_{M,N} = 3$ is the interior of a tangent hexagon.
1953 Moscow Mathematical Olympiad, 243
Given a right circular cone and a point $A$. Find the set of vertices of cones equal to the given one, with axes parallel to that of the given one, and with $A$ inside them. We shall assume that the cone is infinite in one side.